Integrating Partial Fractions with a Twist: (x + 3) vs (x - 3)

Click For Summary
The discussion centers on the integration of partial fractions, specifically addressing the term with 1/(x - 3). Participants highlight that an earlier mistake may have influenced the fractions used in the integration process. There is a significant issue with the limits of integration at x = 3, which cannot be ignored. The correct denominator should be (x + 3) rather than (x - 3), as the latter leads to undefined logarithmic terms. Overall, the integration approach needs reevaluation due to these discrepancies.
maxim07
Messages
53
Reaction score
8
Homework Statement
Question states that integral of given partial fraction is supposed to equal ln(2/sqrt(3)), but I have tried it and it does not seem to. I believe I have integrated correctly, but maybe it can be simplified in such a way to become ln(2/sqrt(2))
Relevant Equations
Partial fraction to integrate = 5/7(2x+1) + 1/7(x-3)
BA7B67FF-82D5-4E6A-9ADA-D5F9C75ACAD2.jpeg
 
Physics news on Phys.org
What happened to the term with ##\frac 1 {x - 3}##?

Are you sure there isn't an earlier mistake that led to those fractions?
 
This is how I go the reactions -pretty sure it’s right
C490592A-7A97-44F8-B669-B4ECA3DDC9A7.jpeg
 
The second term is problem with those limits of integration - in particular at ##x = 3##. You can't make that go away.
 
  • Like
Likes docnet
the denominator of your second integral is (x + 3) not (x - 3) as in the question. The version with (x + 3) is closer to ln(2/sqrt(3) but still differs by about 2.4 x 10^-3.

using (x - 3) does not work since using the limits given the term becomes ln|-1| and ln(0) so maybe there is something wrong with the question
 
Question: A clock's minute hand has length 4 and its hour hand has length 3. What is the distance between the tips at the moment when it is increasing most rapidly?(Putnam Exam Question) Answer: Making assumption that both the hands moves at constant angular velocities, the answer is ## \sqrt{7} .## But don't you think this assumption is somewhat doubtful and wrong?

Similar threads

  • · Replies 8 ·
Replies
8
Views
2K
  • · Replies 16 ·
Replies
16
Views
2K
  • · Replies 1 ·
Replies
1
Views
1K
  • · Replies 6 ·
Replies
6
Views
1K
Replies
9
Views
2K
  • · Replies 4 ·
Replies
4
Views
2K
  • · Replies 6 ·
Replies
6
Views
2K
  • · Replies 8 ·
Replies
8
Views
1K
  • · Replies 1 ·
Replies
1
Views
2K
  • · Replies 6 ·
Replies
6
Views
2K